How to calculate 3d rotation from 3 points?











up vote
5
down vote

favorite
1












If you have 3 labeled points on a surface of a paper. Like



    1


2 3


This makes a perfect equilateral triangle.



From this perspective I can say that the camera is on top of the paper looking down. We can say the camera is at coordinate $(0,0,100)$. Which is 0 degree rotation in Z axis and 90 degree rotation in Y axis.



Then, I move the camera to some arbitrary spot. Now, the points are like



         1

2 3


This looks like the camera is farther back from the paper and was lowered, say at location $(0, -100, 50)$. Which is about -90 degree rotation in Z-axis, and 45 degree rotation in Y axis.



So my question is basically, given the $(x_1,y_1), (x_2,y_2), (x_3,y_3)$. Is there some formula that can take these arbitrary points, compare it with the original 3, to know how much of a X,Y,Z rotation it is of the camera?



I can also rotate on angles like this. For example, I can take the second example from above, then I can rotate my head clockwise, making the numbers flip like



2
1
3


I think getting a normal vector from the center might be better to find.










share|cite|improve this question
























  • Do these points lie on the same plane?
    – johny
    Mar 10 '17 at 3:29










  • yes they do lie on a plane. You can think of it like this. The original 4 lie on a paper thats on a flat surface plane, like I guess z=0. But then you can look at the 4 points from any angle and from any position in 3d space.
    – omega
    Mar 10 '17 at 3:32












  • so you will have to find the rotation and the translation of the camera. That is you are looking for 9+3 parameters right?
    – johny
    Mar 10 '17 at 3:35












  • basically I think so.
    – omega
    Mar 10 '17 at 3:38










  • I also updated it to be 3 points instead of 4 without loss of generality.
    – omega
    Mar 10 '17 at 3:38















up vote
5
down vote

favorite
1












If you have 3 labeled points on a surface of a paper. Like



    1


2 3


This makes a perfect equilateral triangle.



From this perspective I can say that the camera is on top of the paper looking down. We can say the camera is at coordinate $(0,0,100)$. Which is 0 degree rotation in Z axis and 90 degree rotation in Y axis.



Then, I move the camera to some arbitrary spot. Now, the points are like



         1

2 3


This looks like the camera is farther back from the paper and was lowered, say at location $(0, -100, 50)$. Which is about -90 degree rotation in Z-axis, and 45 degree rotation in Y axis.



So my question is basically, given the $(x_1,y_1), (x_2,y_2), (x_3,y_3)$. Is there some formula that can take these arbitrary points, compare it with the original 3, to know how much of a X,Y,Z rotation it is of the camera?



I can also rotate on angles like this. For example, I can take the second example from above, then I can rotate my head clockwise, making the numbers flip like



2
1
3


I think getting a normal vector from the center might be better to find.










share|cite|improve this question
























  • Do these points lie on the same plane?
    – johny
    Mar 10 '17 at 3:29










  • yes they do lie on a plane. You can think of it like this. The original 4 lie on a paper thats on a flat surface plane, like I guess z=0. But then you can look at the 4 points from any angle and from any position in 3d space.
    – omega
    Mar 10 '17 at 3:32












  • so you will have to find the rotation and the translation of the camera. That is you are looking for 9+3 parameters right?
    – johny
    Mar 10 '17 at 3:35












  • basically I think so.
    – omega
    Mar 10 '17 at 3:38










  • I also updated it to be 3 points instead of 4 without loss of generality.
    – omega
    Mar 10 '17 at 3:38













up vote
5
down vote

favorite
1









up vote
5
down vote

favorite
1






1





If you have 3 labeled points on a surface of a paper. Like



    1


2 3


This makes a perfect equilateral triangle.



From this perspective I can say that the camera is on top of the paper looking down. We can say the camera is at coordinate $(0,0,100)$. Which is 0 degree rotation in Z axis and 90 degree rotation in Y axis.



Then, I move the camera to some arbitrary spot. Now, the points are like



         1

2 3


This looks like the camera is farther back from the paper and was lowered, say at location $(0, -100, 50)$. Which is about -90 degree rotation in Z-axis, and 45 degree rotation in Y axis.



So my question is basically, given the $(x_1,y_1), (x_2,y_2), (x_3,y_3)$. Is there some formula that can take these arbitrary points, compare it with the original 3, to know how much of a X,Y,Z rotation it is of the camera?



I can also rotate on angles like this. For example, I can take the second example from above, then I can rotate my head clockwise, making the numbers flip like



2
1
3


I think getting a normal vector from the center might be better to find.










share|cite|improve this question















If you have 3 labeled points on a surface of a paper. Like



    1


2 3


This makes a perfect equilateral triangle.



From this perspective I can say that the camera is on top of the paper looking down. We can say the camera is at coordinate $(0,0,100)$. Which is 0 degree rotation in Z axis and 90 degree rotation in Y axis.



Then, I move the camera to some arbitrary spot. Now, the points are like



         1

2 3


This looks like the camera is farther back from the paper and was lowered, say at location $(0, -100, 50)$. Which is about -90 degree rotation in Z-axis, and 45 degree rotation in Y axis.



So my question is basically, given the $(x_1,y_1), (x_2,y_2), (x_3,y_3)$. Is there some formula that can take these arbitrary points, compare it with the original 3, to know how much of a X,Y,Z rotation it is of the camera?



I can also rotate on angles like this. For example, I can take the second example from above, then I can rotate my head clockwise, making the numbers flip like



2
1
3


I think getting a normal vector from the center might be better to find.







linear-algebra geometry rotations






share|cite|improve this question















share|cite|improve this question













share|cite|improve this question




share|cite|improve this question








edited Mar 10 '17 at 3:40

























asked Mar 10 '17 at 3:05









omega

389217




389217












  • Do these points lie on the same plane?
    – johny
    Mar 10 '17 at 3:29










  • yes they do lie on a plane. You can think of it like this. The original 4 lie on a paper thats on a flat surface plane, like I guess z=0. But then you can look at the 4 points from any angle and from any position in 3d space.
    – omega
    Mar 10 '17 at 3:32












  • so you will have to find the rotation and the translation of the camera. That is you are looking for 9+3 parameters right?
    – johny
    Mar 10 '17 at 3:35












  • basically I think so.
    – omega
    Mar 10 '17 at 3:38










  • I also updated it to be 3 points instead of 4 without loss of generality.
    – omega
    Mar 10 '17 at 3:38


















  • Do these points lie on the same plane?
    – johny
    Mar 10 '17 at 3:29










  • yes they do lie on a plane. You can think of it like this. The original 4 lie on a paper thats on a flat surface plane, like I guess z=0. But then you can look at the 4 points from any angle and from any position in 3d space.
    – omega
    Mar 10 '17 at 3:32












  • so you will have to find the rotation and the translation of the camera. That is you are looking for 9+3 parameters right?
    – johny
    Mar 10 '17 at 3:35












  • basically I think so.
    – omega
    Mar 10 '17 at 3:38










  • I also updated it to be 3 points instead of 4 without loss of generality.
    – omega
    Mar 10 '17 at 3:38
















Do these points lie on the same plane?
– johny
Mar 10 '17 at 3:29




Do these points lie on the same plane?
– johny
Mar 10 '17 at 3:29












yes they do lie on a plane. You can think of it like this. The original 4 lie on a paper thats on a flat surface plane, like I guess z=0. But then you can look at the 4 points from any angle and from any position in 3d space.
– omega
Mar 10 '17 at 3:32






yes they do lie on a plane. You can think of it like this. The original 4 lie on a paper thats on a flat surface plane, like I guess z=0. But then you can look at the 4 points from any angle and from any position in 3d space.
– omega
Mar 10 '17 at 3:32














so you will have to find the rotation and the translation of the camera. That is you are looking for 9+3 parameters right?
– johny
Mar 10 '17 at 3:35






so you will have to find the rotation and the translation of the camera. That is you are looking for 9+3 parameters right?
– johny
Mar 10 '17 at 3:35














basically I think so.
– omega
Mar 10 '17 at 3:38




basically I think so.
– omega
Mar 10 '17 at 3:38












I also updated it to be 3 points instead of 4 without loss of generality.
– omega
Mar 10 '17 at 3:38




I also updated it to be 3 points instead of 4 without loss of generality.
– omega
Mar 10 '17 at 3:38










2 Answers
2






active

oldest

votes

















up vote
0
down vote













Assumption:

I am assuming that the camera is orthographic, since for a perspective projection you would need at least 4 points to determine the position/rotation of the camera.



General Overview of the Method:

Read about the camera transformation here -> https://en.wikipedia.org/wiki/3D_projection#Perspective_projection .



Let the camera transform matrix be $C$. Let the original point be $p$ and the projected point be $b$. Then the orthographic projection is simply $(b_x, b_y)$ where
$$ begin{bmatrix}
b_x\
b_y\
b_z
end{bmatrix}=Ccdot p $$



Let $v_{1,2,3}$ be the vertices of the original triangle. Let $b_{1,2,3}$ be the vertices of the triangle in the transformed coordinate space.
$$Ccdot [v_1 v_2 v_3]=[b_1 b_2 b_3]$$
Thus
$$C=[b_1 b_2 b_3]cdot [v_1 v_2 v_3]^{-1}$$



If you fill in values for $v_{1,2,3}$ and only the $x$ and $y$ coordinates of $b_{1,2,3}$, this should leave you with a matrix with some unknowns ($b_{1z}$, $b_{2z}$, and $b_{3z}$), but from there you can deconstruct the camera transform into rotation (and translation), if you make the assumption that the camera is looking at the origin, and that the camera is a fixed distance away.



Good luck. This should be enough to get you started, but I will expand this answer if you need it.






share|cite|improve this answer























  • It's a bit confusing, Is v1,v2,v3 3 coordinates or 2 coordinates? The input is just 3 2d coordinates for the original triangle, and 3 2d coordinates for the other triangle. And why is z=0 for b? Maybe it would be clearer if you could expand all the vectors fully.
    – omega
    Mar 10 '17 at 4:47










  • And yes, we can say the center of the original triangle can be the origin (0,0,0).
    – omega
    Mar 10 '17 at 4:54


















up vote
0
down vote













Let, as a model, we use a camera point $K$ and a camera screen $k$, where $k$ is a plane, and let the orthogonal projection $O$ of point $K$ on the plane $k$ be the origin of the two dimensional coordinate system $Oe_1e_2$ on the screen $k$. Attach a three dimensional coordinate system $Ke_1e_2e_3$ to the point $K$ where $e_1$ and $e_2$ are the unit vectors of $Oe_1e_2$ and let $e_3$ be the unit vector orthogonal to $k$ and pointing from $K$ to $k$. Denote by $nu$ the distance from $K$ to $k$. We may or may not know it.



At first let $A_0B_0C_0$ be an equilateral triangle in a plane $p$ parallel to $k$. Let $G_0 in p$ be the center of $A_0B_0C_0$ so that $G_0$ lies on the $e_3$ axis, i.e. $G_0$ is the orthogonal projection of $K$ onto $p$. Then we project $A_0B_0C_0$ onto the plane $k$ by drawing the lines $KA_0, , KB_0, , KC_0$ and possibly $KG_0$ and denoting by $A, B, C$ and $O$ their intersection points with $k$. Thus we obtain the projected equilateral triangle $ABC$ with center $O$.



Next we move the camera $K$ together with the screen $k$ to a different position and again we project the triangle $A_0B_0C_0$ with center $G_0$ onto the plane $k$ (which is now positioned differently with respect to $p$ and is this possibly not parallel to it) the same way we did before. We draw the lines between point $K$ and the points $A_0, B_0, C_0$ and $G_0$ and look at their respective intersection points $A, B, C$ and $G$ with plane $k$. The resulting triangle $ABC$ is most likely not equilateral.



Think of the coordinate system $Ke_1e_2e_3$, which moved together with the camera and the screen, and recall that $nu$ is the distance between $K$ and $k$, the orthogonal projection of $K$ onto $k$ is $O$, and $Oe_1e_2$ is the coordinate system on the screen $k$ as before.



If we take a plane $q$ parallel to $p$ passing through point say $C$, then the intersection points
$$A_q = KA cap q, ,, B_q = KB cap q, ,, C_q = KC cap q , text{ and } , G_q = KG cap q$$
form an equilateral triangle $A_qB_qC$ with center $G_q$ whose edges are parallel to the edges of $A_0B_0C_0$ (in other words triangle $A_qB_qC$ with center $G_q$ is homothetic image of triangle $A_0B_0C_0$ with center $G_0$ with respect $K$ )



Now, you know the locations of the points $A, , B, , C$ and say $G$ with respect to the screen's $k$ coordinate system $Oe_1e_2$ which means you know the vectors $$vec{OA}, , vec{OB}, , vec{OC}, ,, text{ and } ,, vec{OG}$$ so thus, we can form the vectors
begin{align*}
vec{KA} &= vec{KO} + vec{OA} = vec{OA} + vec{KO} = vec{OA} + nu , e_3\
vec{KB} &= vec{KO} + vec{OB} = vec{OB} + vec{KO} = vec{OB} + nu , e_3\
vec{KC} &= vec{KO} + vec{OC} = vec{OC} + vec{KO} = vec{OC} + nu , e_3\
vec{KG} &= vec{KO} + vec{OG} = vec{OG} + vec{KO} = vec{OG} + nu , e_3\
end{align*}

Thus, there exists constants $lambda, , mu$ and $nu$, if you do not know $nu$
begin{align*}
vec{KA_q} &= lambda , vec{KA}= lambda , vec{OA} + lambda , nu , e_3\
vec{KB_q} &= mu , vec{KB}= mu , vec{OA} + mu , nu , e_3\
vec{KG_q} &= frac{1}{3} big(lambda , vec{KA} + mu , vec{KB} + vec{KC}big)\ &= frac{1}{3} big( lambda , vec{OA} + mu , vec{OA} + vec{OC} + (lambda + mu + 1) , nu , e_3 big)
end{align*}
The restrictions that lead to the equations for the unknowns $lambda, , mu$ and possibly $nu$ are
begin{align*}
|vec{C_qA_q}| &= |vec{A_qB_q}|\
|vec{C_qB_q}| &= |vec{A_qB_q}|\
vec{KG_q} & , |, | , vec{KG}\
end{align*} where the first two say that the edges of equilateral triangle $A_qB_qC$ are equal while the last one represents the fact that the points $K, , G_q $ and $G$ are collinear. As equations, these three conditions are



begin{align*}
|lambda , vec{KA} - vec{KC}|^2 &= |lambda , vec{KA} - mu , vec{KB}|^2 \
|lambda , vec{KA} - vec{KC}|^2 &= |lambda , vec{KA} - mu , vec{KB}|^2 \
|vec{KG_q}times vec{KG}|^2 &= 0\
end{align*}
which written in terms of $lambda, , mu, , nu$ unknown variables turn into



begin{align*}
|lambda , vec{OA} - vec{OC} + (lambda , nu , - nu) ,
e_3 |^2 &= |lambda , vec{OA} - mu , vec{OB} + (lambda ,
nu , - mu , nu) ,
e_3 |^2 \
|mu , vec{OB} - vec{OC} + (mu , nu , - nu) , e_3 |^2 &=
|lambda , vec{OA} - mu , vec{OB} + (lambda , nu , - mu
, nu) ,
e_3 |^2 \
|big(frac{1}{3} big( lambda , vec{OA} + mu ,
vec{OA} + vec{OC} + (lambda + mu + 1) , nu , e_3 big)big)
times big(vec{OG} + nu , e_3 big) |^2 &= 0\
end{align*}



If you know $nu$ then only the first two equations are relevant



begin{align*}
|lambda , vec{OA} - vec{OC} + (lambda , nu , - nu) ,
e_3 |^2 &= |lambda , vec{OA} - mu , vec{OB} + (lambda ,
nu , - mu , nu) ,
e_3 |^2 \
|mu , vec{OB} - vec{OC} + (mu , nu , - nu) , e_3 |^2 &=
|lambda , vec{OA} - mu , vec{OB} + (lambda , nu , - mu
, nu) ,
e_3 |^2
end{align*}



All these systems of equations may have several solutions though.



I haven't checked for typos and mistakes and I do not guarantee this is the best solution.






share|cite|improve this answer





















    Your Answer





    StackExchange.ifUsing("editor", function () {
    return StackExchange.using("mathjaxEditing", function () {
    StackExchange.MarkdownEditor.creationCallbacks.add(function (editor, postfix) {
    StackExchange.mathjaxEditing.prepareWmdForMathJax(editor, postfix, [["$", "$"], ["\\(","\\)"]]);
    });
    });
    }, "mathjax-editing");

    StackExchange.ready(function() {
    var channelOptions = {
    tags: "".split(" "),
    id: "69"
    };
    initTagRenderer("".split(" "), "".split(" "), channelOptions);

    StackExchange.using("externalEditor", function() {
    // Have to fire editor after snippets, if snippets enabled
    if (StackExchange.settings.snippets.snippetsEnabled) {
    StackExchange.using("snippets", function() {
    createEditor();
    });
    }
    else {
    createEditor();
    }
    });

    function createEditor() {
    StackExchange.prepareEditor({
    heartbeatType: 'answer',
    convertImagesToLinks: true,
    noModals: true,
    showLowRepImageUploadWarning: true,
    reputationToPostImages: 10,
    bindNavPrevention: true,
    postfix: "",
    imageUploader: {
    brandingHtml: "Powered by u003ca class="icon-imgur-white" href="https://imgur.com/"u003eu003c/au003e",
    contentPolicyHtml: "User contributions licensed under u003ca href="https://creativecommons.org/licenses/by-sa/3.0/"u003ecc by-sa 3.0 with attribution requiredu003c/au003e u003ca href="https://stackoverflow.com/legal/content-policy"u003e(content policy)u003c/au003e",
    allowUrls: true
    },
    noCode: true, onDemand: true,
    discardSelector: ".discard-answer"
    ,immediatelyShowMarkdownHelp:true
    });


    }
    });














    draft saved

    draft discarded


















    StackExchange.ready(
    function () {
    StackExchange.openid.initPostLogin('.new-post-login', 'https%3a%2f%2fmath.stackexchange.com%2fquestions%2f2180102%2fhow-to-calculate-3d-rotation-from-3-points%23new-answer', 'question_page');
    }
    );

    Post as a guest















    Required, but never shown

























    2 Answers
    2






    active

    oldest

    votes








    2 Answers
    2






    active

    oldest

    votes









    active

    oldest

    votes






    active

    oldest

    votes








    up vote
    0
    down vote













    Assumption:

    I am assuming that the camera is orthographic, since for a perspective projection you would need at least 4 points to determine the position/rotation of the camera.



    General Overview of the Method:

    Read about the camera transformation here -> https://en.wikipedia.org/wiki/3D_projection#Perspective_projection .



    Let the camera transform matrix be $C$. Let the original point be $p$ and the projected point be $b$. Then the orthographic projection is simply $(b_x, b_y)$ where
    $$ begin{bmatrix}
    b_x\
    b_y\
    b_z
    end{bmatrix}=Ccdot p $$



    Let $v_{1,2,3}$ be the vertices of the original triangle. Let $b_{1,2,3}$ be the vertices of the triangle in the transformed coordinate space.
    $$Ccdot [v_1 v_2 v_3]=[b_1 b_2 b_3]$$
    Thus
    $$C=[b_1 b_2 b_3]cdot [v_1 v_2 v_3]^{-1}$$



    If you fill in values for $v_{1,2,3}$ and only the $x$ and $y$ coordinates of $b_{1,2,3}$, this should leave you with a matrix with some unknowns ($b_{1z}$, $b_{2z}$, and $b_{3z}$), but from there you can deconstruct the camera transform into rotation (and translation), if you make the assumption that the camera is looking at the origin, and that the camera is a fixed distance away.



    Good luck. This should be enough to get you started, but I will expand this answer if you need it.






    share|cite|improve this answer























    • It's a bit confusing, Is v1,v2,v3 3 coordinates or 2 coordinates? The input is just 3 2d coordinates for the original triangle, and 3 2d coordinates for the other triangle. And why is z=0 for b? Maybe it would be clearer if you could expand all the vectors fully.
      – omega
      Mar 10 '17 at 4:47










    • And yes, we can say the center of the original triangle can be the origin (0,0,0).
      – omega
      Mar 10 '17 at 4:54















    up vote
    0
    down vote













    Assumption:

    I am assuming that the camera is orthographic, since for a perspective projection you would need at least 4 points to determine the position/rotation of the camera.



    General Overview of the Method:

    Read about the camera transformation here -> https://en.wikipedia.org/wiki/3D_projection#Perspective_projection .



    Let the camera transform matrix be $C$. Let the original point be $p$ and the projected point be $b$. Then the orthographic projection is simply $(b_x, b_y)$ where
    $$ begin{bmatrix}
    b_x\
    b_y\
    b_z
    end{bmatrix}=Ccdot p $$



    Let $v_{1,2,3}$ be the vertices of the original triangle. Let $b_{1,2,3}$ be the vertices of the triangle in the transformed coordinate space.
    $$Ccdot [v_1 v_2 v_3]=[b_1 b_2 b_3]$$
    Thus
    $$C=[b_1 b_2 b_3]cdot [v_1 v_2 v_3]^{-1}$$



    If you fill in values for $v_{1,2,3}$ and only the $x$ and $y$ coordinates of $b_{1,2,3}$, this should leave you with a matrix with some unknowns ($b_{1z}$, $b_{2z}$, and $b_{3z}$), but from there you can deconstruct the camera transform into rotation (and translation), if you make the assumption that the camera is looking at the origin, and that the camera is a fixed distance away.



    Good luck. This should be enough to get you started, but I will expand this answer if you need it.






    share|cite|improve this answer























    • It's a bit confusing, Is v1,v2,v3 3 coordinates or 2 coordinates? The input is just 3 2d coordinates for the original triangle, and 3 2d coordinates for the other triangle. And why is z=0 for b? Maybe it would be clearer if you could expand all the vectors fully.
      – omega
      Mar 10 '17 at 4:47










    • And yes, we can say the center of the original triangle can be the origin (0,0,0).
      – omega
      Mar 10 '17 at 4:54













    up vote
    0
    down vote










    up vote
    0
    down vote









    Assumption:

    I am assuming that the camera is orthographic, since for a perspective projection you would need at least 4 points to determine the position/rotation of the camera.



    General Overview of the Method:

    Read about the camera transformation here -> https://en.wikipedia.org/wiki/3D_projection#Perspective_projection .



    Let the camera transform matrix be $C$. Let the original point be $p$ and the projected point be $b$. Then the orthographic projection is simply $(b_x, b_y)$ where
    $$ begin{bmatrix}
    b_x\
    b_y\
    b_z
    end{bmatrix}=Ccdot p $$



    Let $v_{1,2,3}$ be the vertices of the original triangle. Let $b_{1,2,3}$ be the vertices of the triangle in the transformed coordinate space.
    $$Ccdot [v_1 v_2 v_3]=[b_1 b_2 b_3]$$
    Thus
    $$C=[b_1 b_2 b_3]cdot [v_1 v_2 v_3]^{-1}$$



    If you fill in values for $v_{1,2,3}$ and only the $x$ and $y$ coordinates of $b_{1,2,3}$, this should leave you with a matrix with some unknowns ($b_{1z}$, $b_{2z}$, and $b_{3z}$), but from there you can deconstruct the camera transform into rotation (and translation), if you make the assumption that the camera is looking at the origin, and that the camera is a fixed distance away.



    Good luck. This should be enough to get you started, but I will expand this answer if you need it.






    share|cite|improve this answer














    Assumption:

    I am assuming that the camera is orthographic, since for a perspective projection you would need at least 4 points to determine the position/rotation of the camera.



    General Overview of the Method:

    Read about the camera transformation here -> https://en.wikipedia.org/wiki/3D_projection#Perspective_projection .



    Let the camera transform matrix be $C$. Let the original point be $p$ and the projected point be $b$. Then the orthographic projection is simply $(b_x, b_y)$ where
    $$ begin{bmatrix}
    b_x\
    b_y\
    b_z
    end{bmatrix}=Ccdot p $$



    Let $v_{1,2,3}$ be the vertices of the original triangle. Let $b_{1,2,3}$ be the vertices of the triangle in the transformed coordinate space.
    $$Ccdot [v_1 v_2 v_3]=[b_1 b_2 b_3]$$
    Thus
    $$C=[b_1 b_2 b_3]cdot [v_1 v_2 v_3]^{-1}$$



    If you fill in values for $v_{1,2,3}$ and only the $x$ and $y$ coordinates of $b_{1,2,3}$, this should leave you with a matrix with some unknowns ($b_{1z}$, $b_{2z}$, and $b_{3z}$), but from there you can deconstruct the camera transform into rotation (and translation), if you make the assumption that the camera is looking at the origin, and that the camera is a fixed distance away.



    Good luck. This should be enough to get you started, but I will expand this answer if you need it.







    share|cite|improve this answer














    share|cite|improve this answer



    share|cite|improve this answer








    edited Mar 10 '17 at 4:58

























    answered Mar 10 '17 at 4:42









    KoreanwGlasses

    1083




    1083












    • It's a bit confusing, Is v1,v2,v3 3 coordinates or 2 coordinates? The input is just 3 2d coordinates for the original triangle, and 3 2d coordinates for the other triangle. And why is z=0 for b? Maybe it would be clearer if you could expand all the vectors fully.
      – omega
      Mar 10 '17 at 4:47










    • And yes, we can say the center of the original triangle can be the origin (0,0,0).
      – omega
      Mar 10 '17 at 4:54


















    • It's a bit confusing, Is v1,v2,v3 3 coordinates or 2 coordinates? The input is just 3 2d coordinates for the original triangle, and 3 2d coordinates for the other triangle. And why is z=0 for b? Maybe it would be clearer if you could expand all the vectors fully.
      – omega
      Mar 10 '17 at 4:47










    • And yes, we can say the center of the original triangle can be the origin (0,0,0).
      – omega
      Mar 10 '17 at 4:54
















    It's a bit confusing, Is v1,v2,v3 3 coordinates or 2 coordinates? The input is just 3 2d coordinates for the original triangle, and 3 2d coordinates for the other triangle. And why is z=0 for b? Maybe it would be clearer if you could expand all the vectors fully.
    – omega
    Mar 10 '17 at 4:47




    It's a bit confusing, Is v1,v2,v3 3 coordinates or 2 coordinates? The input is just 3 2d coordinates for the original triangle, and 3 2d coordinates for the other triangle. And why is z=0 for b? Maybe it would be clearer if you could expand all the vectors fully.
    – omega
    Mar 10 '17 at 4:47












    And yes, we can say the center of the original triangle can be the origin (0,0,0).
    – omega
    Mar 10 '17 at 4:54




    And yes, we can say the center of the original triangle can be the origin (0,0,0).
    – omega
    Mar 10 '17 at 4:54










    up vote
    0
    down vote













    Let, as a model, we use a camera point $K$ and a camera screen $k$, where $k$ is a plane, and let the orthogonal projection $O$ of point $K$ on the plane $k$ be the origin of the two dimensional coordinate system $Oe_1e_2$ on the screen $k$. Attach a three dimensional coordinate system $Ke_1e_2e_3$ to the point $K$ where $e_1$ and $e_2$ are the unit vectors of $Oe_1e_2$ and let $e_3$ be the unit vector orthogonal to $k$ and pointing from $K$ to $k$. Denote by $nu$ the distance from $K$ to $k$. We may or may not know it.



    At first let $A_0B_0C_0$ be an equilateral triangle in a plane $p$ parallel to $k$. Let $G_0 in p$ be the center of $A_0B_0C_0$ so that $G_0$ lies on the $e_3$ axis, i.e. $G_0$ is the orthogonal projection of $K$ onto $p$. Then we project $A_0B_0C_0$ onto the plane $k$ by drawing the lines $KA_0, , KB_0, , KC_0$ and possibly $KG_0$ and denoting by $A, B, C$ and $O$ their intersection points with $k$. Thus we obtain the projected equilateral triangle $ABC$ with center $O$.



    Next we move the camera $K$ together with the screen $k$ to a different position and again we project the triangle $A_0B_0C_0$ with center $G_0$ onto the plane $k$ (which is now positioned differently with respect to $p$ and is this possibly not parallel to it) the same way we did before. We draw the lines between point $K$ and the points $A_0, B_0, C_0$ and $G_0$ and look at their respective intersection points $A, B, C$ and $G$ with plane $k$. The resulting triangle $ABC$ is most likely not equilateral.



    Think of the coordinate system $Ke_1e_2e_3$, which moved together with the camera and the screen, and recall that $nu$ is the distance between $K$ and $k$, the orthogonal projection of $K$ onto $k$ is $O$, and $Oe_1e_2$ is the coordinate system on the screen $k$ as before.



    If we take a plane $q$ parallel to $p$ passing through point say $C$, then the intersection points
    $$A_q = KA cap q, ,, B_q = KB cap q, ,, C_q = KC cap q , text{ and } , G_q = KG cap q$$
    form an equilateral triangle $A_qB_qC$ with center $G_q$ whose edges are parallel to the edges of $A_0B_0C_0$ (in other words triangle $A_qB_qC$ with center $G_q$ is homothetic image of triangle $A_0B_0C_0$ with center $G_0$ with respect $K$ )



    Now, you know the locations of the points $A, , B, , C$ and say $G$ with respect to the screen's $k$ coordinate system $Oe_1e_2$ which means you know the vectors $$vec{OA}, , vec{OB}, , vec{OC}, ,, text{ and } ,, vec{OG}$$ so thus, we can form the vectors
    begin{align*}
    vec{KA} &= vec{KO} + vec{OA} = vec{OA} + vec{KO} = vec{OA} + nu , e_3\
    vec{KB} &= vec{KO} + vec{OB} = vec{OB} + vec{KO} = vec{OB} + nu , e_3\
    vec{KC} &= vec{KO} + vec{OC} = vec{OC} + vec{KO} = vec{OC} + nu , e_3\
    vec{KG} &= vec{KO} + vec{OG} = vec{OG} + vec{KO} = vec{OG} + nu , e_3\
    end{align*}

    Thus, there exists constants $lambda, , mu$ and $nu$, if you do not know $nu$
    begin{align*}
    vec{KA_q} &= lambda , vec{KA}= lambda , vec{OA} + lambda , nu , e_3\
    vec{KB_q} &= mu , vec{KB}= mu , vec{OA} + mu , nu , e_3\
    vec{KG_q} &= frac{1}{3} big(lambda , vec{KA} + mu , vec{KB} + vec{KC}big)\ &= frac{1}{3} big( lambda , vec{OA} + mu , vec{OA} + vec{OC} + (lambda + mu + 1) , nu , e_3 big)
    end{align*}
    The restrictions that lead to the equations for the unknowns $lambda, , mu$ and possibly $nu$ are
    begin{align*}
    |vec{C_qA_q}| &= |vec{A_qB_q}|\
    |vec{C_qB_q}| &= |vec{A_qB_q}|\
    vec{KG_q} & , |, | , vec{KG}\
    end{align*} where the first two say that the edges of equilateral triangle $A_qB_qC$ are equal while the last one represents the fact that the points $K, , G_q $ and $G$ are collinear. As equations, these three conditions are



    begin{align*}
    |lambda , vec{KA} - vec{KC}|^2 &= |lambda , vec{KA} - mu , vec{KB}|^2 \
    |lambda , vec{KA} - vec{KC}|^2 &= |lambda , vec{KA} - mu , vec{KB}|^2 \
    |vec{KG_q}times vec{KG}|^2 &= 0\
    end{align*}
    which written in terms of $lambda, , mu, , nu$ unknown variables turn into



    begin{align*}
    |lambda , vec{OA} - vec{OC} + (lambda , nu , - nu) ,
    e_3 |^2 &= |lambda , vec{OA} - mu , vec{OB} + (lambda ,
    nu , - mu , nu) ,
    e_3 |^2 \
    |mu , vec{OB} - vec{OC} + (mu , nu , - nu) , e_3 |^2 &=
    |lambda , vec{OA} - mu , vec{OB} + (lambda , nu , - mu
    , nu) ,
    e_3 |^2 \
    |big(frac{1}{3} big( lambda , vec{OA} + mu ,
    vec{OA} + vec{OC} + (lambda + mu + 1) , nu , e_3 big)big)
    times big(vec{OG} + nu , e_3 big) |^2 &= 0\
    end{align*}



    If you know $nu$ then only the first two equations are relevant



    begin{align*}
    |lambda , vec{OA} - vec{OC} + (lambda , nu , - nu) ,
    e_3 |^2 &= |lambda , vec{OA} - mu , vec{OB} + (lambda ,
    nu , - mu , nu) ,
    e_3 |^2 \
    |mu , vec{OB} - vec{OC} + (mu , nu , - nu) , e_3 |^2 &=
    |lambda , vec{OA} - mu , vec{OB} + (lambda , nu , - mu
    , nu) ,
    e_3 |^2
    end{align*}



    All these systems of equations may have several solutions though.



    I haven't checked for typos and mistakes and I do not guarantee this is the best solution.






    share|cite|improve this answer

























      up vote
      0
      down vote













      Let, as a model, we use a camera point $K$ and a camera screen $k$, where $k$ is a plane, and let the orthogonal projection $O$ of point $K$ on the plane $k$ be the origin of the two dimensional coordinate system $Oe_1e_2$ on the screen $k$. Attach a three dimensional coordinate system $Ke_1e_2e_3$ to the point $K$ where $e_1$ and $e_2$ are the unit vectors of $Oe_1e_2$ and let $e_3$ be the unit vector orthogonal to $k$ and pointing from $K$ to $k$. Denote by $nu$ the distance from $K$ to $k$. We may or may not know it.



      At first let $A_0B_0C_0$ be an equilateral triangle in a plane $p$ parallel to $k$. Let $G_0 in p$ be the center of $A_0B_0C_0$ so that $G_0$ lies on the $e_3$ axis, i.e. $G_0$ is the orthogonal projection of $K$ onto $p$. Then we project $A_0B_0C_0$ onto the plane $k$ by drawing the lines $KA_0, , KB_0, , KC_0$ and possibly $KG_0$ and denoting by $A, B, C$ and $O$ their intersection points with $k$. Thus we obtain the projected equilateral triangle $ABC$ with center $O$.



      Next we move the camera $K$ together with the screen $k$ to a different position and again we project the triangle $A_0B_0C_0$ with center $G_0$ onto the plane $k$ (which is now positioned differently with respect to $p$ and is this possibly not parallel to it) the same way we did before. We draw the lines between point $K$ and the points $A_0, B_0, C_0$ and $G_0$ and look at their respective intersection points $A, B, C$ and $G$ with plane $k$. The resulting triangle $ABC$ is most likely not equilateral.



      Think of the coordinate system $Ke_1e_2e_3$, which moved together with the camera and the screen, and recall that $nu$ is the distance between $K$ and $k$, the orthogonal projection of $K$ onto $k$ is $O$, and $Oe_1e_2$ is the coordinate system on the screen $k$ as before.



      If we take a plane $q$ parallel to $p$ passing through point say $C$, then the intersection points
      $$A_q = KA cap q, ,, B_q = KB cap q, ,, C_q = KC cap q , text{ and } , G_q = KG cap q$$
      form an equilateral triangle $A_qB_qC$ with center $G_q$ whose edges are parallel to the edges of $A_0B_0C_0$ (in other words triangle $A_qB_qC$ with center $G_q$ is homothetic image of triangle $A_0B_0C_0$ with center $G_0$ with respect $K$ )



      Now, you know the locations of the points $A, , B, , C$ and say $G$ with respect to the screen's $k$ coordinate system $Oe_1e_2$ which means you know the vectors $$vec{OA}, , vec{OB}, , vec{OC}, ,, text{ and } ,, vec{OG}$$ so thus, we can form the vectors
      begin{align*}
      vec{KA} &= vec{KO} + vec{OA} = vec{OA} + vec{KO} = vec{OA} + nu , e_3\
      vec{KB} &= vec{KO} + vec{OB} = vec{OB} + vec{KO} = vec{OB} + nu , e_3\
      vec{KC} &= vec{KO} + vec{OC} = vec{OC} + vec{KO} = vec{OC} + nu , e_3\
      vec{KG} &= vec{KO} + vec{OG} = vec{OG} + vec{KO} = vec{OG} + nu , e_3\
      end{align*}

      Thus, there exists constants $lambda, , mu$ and $nu$, if you do not know $nu$
      begin{align*}
      vec{KA_q} &= lambda , vec{KA}= lambda , vec{OA} + lambda , nu , e_3\
      vec{KB_q} &= mu , vec{KB}= mu , vec{OA} + mu , nu , e_3\
      vec{KG_q} &= frac{1}{3} big(lambda , vec{KA} + mu , vec{KB} + vec{KC}big)\ &= frac{1}{3} big( lambda , vec{OA} + mu , vec{OA} + vec{OC} + (lambda + mu + 1) , nu , e_3 big)
      end{align*}
      The restrictions that lead to the equations for the unknowns $lambda, , mu$ and possibly $nu$ are
      begin{align*}
      |vec{C_qA_q}| &= |vec{A_qB_q}|\
      |vec{C_qB_q}| &= |vec{A_qB_q}|\
      vec{KG_q} & , |, | , vec{KG}\
      end{align*} where the first two say that the edges of equilateral triangle $A_qB_qC$ are equal while the last one represents the fact that the points $K, , G_q $ and $G$ are collinear. As equations, these three conditions are



      begin{align*}
      |lambda , vec{KA} - vec{KC}|^2 &= |lambda , vec{KA} - mu , vec{KB}|^2 \
      |lambda , vec{KA} - vec{KC}|^2 &= |lambda , vec{KA} - mu , vec{KB}|^2 \
      |vec{KG_q}times vec{KG}|^2 &= 0\
      end{align*}
      which written in terms of $lambda, , mu, , nu$ unknown variables turn into



      begin{align*}
      |lambda , vec{OA} - vec{OC} + (lambda , nu , - nu) ,
      e_3 |^2 &= |lambda , vec{OA} - mu , vec{OB} + (lambda ,
      nu , - mu , nu) ,
      e_3 |^2 \
      |mu , vec{OB} - vec{OC} + (mu , nu , - nu) , e_3 |^2 &=
      |lambda , vec{OA} - mu , vec{OB} + (lambda , nu , - mu
      , nu) ,
      e_3 |^2 \
      |big(frac{1}{3} big( lambda , vec{OA} + mu ,
      vec{OA} + vec{OC} + (lambda + mu + 1) , nu , e_3 big)big)
      times big(vec{OG} + nu , e_3 big) |^2 &= 0\
      end{align*}



      If you know $nu$ then only the first two equations are relevant



      begin{align*}
      |lambda , vec{OA} - vec{OC} + (lambda , nu , - nu) ,
      e_3 |^2 &= |lambda , vec{OA} - mu , vec{OB} + (lambda ,
      nu , - mu , nu) ,
      e_3 |^2 \
      |mu , vec{OB} - vec{OC} + (mu , nu , - nu) , e_3 |^2 &=
      |lambda , vec{OA} - mu , vec{OB} + (lambda , nu , - mu
      , nu) ,
      e_3 |^2
      end{align*}



      All these systems of equations may have several solutions though.



      I haven't checked for typos and mistakes and I do not guarantee this is the best solution.






      share|cite|improve this answer























        up vote
        0
        down vote










        up vote
        0
        down vote









        Let, as a model, we use a camera point $K$ and a camera screen $k$, where $k$ is a plane, and let the orthogonal projection $O$ of point $K$ on the plane $k$ be the origin of the two dimensional coordinate system $Oe_1e_2$ on the screen $k$. Attach a three dimensional coordinate system $Ke_1e_2e_3$ to the point $K$ where $e_1$ and $e_2$ are the unit vectors of $Oe_1e_2$ and let $e_3$ be the unit vector orthogonal to $k$ and pointing from $K$ to $k$. Denote by $nu$ the distance from $K$ to $k$. We may or may not know it.



        At first let $A_0B_0C_0$ be an equilateral triangle in a plane $p$ parallel to $k$. Let $G_0 in p$ be the center of $A_0B_0C_0$ so that $G_0$ lies on the $e_3$ axis, i.e. $G_0$ is the orthogonal projection of $K$ onto $p$. Then we project $A_0B_0C_0$ onto the plane $k$ by drawing the lines $KA_0, , KB_0, , KC_0$ and possibly $KG_0$ and denoting by $A, B, C$ and $O$ their intersection points with $k$. Thus we obtain the projected equilateral triangle $ABC$ with center $O$.



        Next we move the camera $K$ together with the screen $k$ to a different position and again we project the triangle $A_0B_0C_0$ with center $G_0$ onto the plane $k$ (which is now positioned differently with respect to $p$ and is this possibly not parallel to it) the same way we did before. We draw the lines between point $K$ and the points $A_0, B_0, C_0$ and $G_0$ and look at their respective intersection points $A, B, C$ and $G$ with plane $k$. The resulting triangle $ABC$ is most likely not equilateral.



        Think of the coordinate system $Ke_1e_2e_3$, which moved together with the camera and the screen, and recall that $nu$ is the distance between $K$ and $k$, the orthogonal projection of $K$ onto $k$ is $O$, and $Oe_1e_2$ is the coordinate system on the screen $k$ as before.



        If we take a plane $q$ parallel to $p$ passing through point say $C$, then the intersection points
        $$A_q = KA cap q, ,, B_q = KB cap q, ,, C_q = KC cap q , text{ and } , G_q = KG cap q$$
        form an equilateral triangle $A_qB_qC$ with center $G_q$ whose edges are parallel to the edges of $A_0B_0C_0$ (in other words triangle $A_qB_qC$ with center $G_q$ is homothetic image of triangle $A_0B_0C_0$ with center $G_0$ with respect $K$ )



        Now, you know the locations of the points $A, , B, , C$ and say $G$ with respect to the screen's $k$ coordinate system $Oe_1e_2$ which means you know the vectors $$vec{OA}, , vec{OB}, , vec{OC}, ,, text{ and } ,, vec{OG}$$ so thus, we can form the vectors
        begin{align*}
        vec{KA} &= vec{KO} + vec{OA} = vec{OA} + vec{KO} = vec{OA} + nu , e_3\
        vec{KB} &= vec{KO} + vec{OB} = vec{OB} + vec{KO} = vec{OB} + nu , e_3\
        vec{KC} &= vec{KO} + vec{OC} = vec{OC} + vec{KO} = vec{OC} + nu , e_3\
        vec{KG} &= vec{KO} + vec{OG} = vec{OG} + vec{KO} = vec{OG} + nu , e_3\
        end{align*}

        Thus, there exists constants $lambda, , mu$ and $nu$, if you do not know $nu$
        begin{align*}
        vec{KA_q} &= lambda , vec{KA}= lambda , vec{OA} + lambda , nu , e_3\
        vec{KB_q} &= mu , vec{KB}= mu , vec{OA} + mu , nu , e_3\
        vec{KG_q} &= frac{1}{3} big(lambda , vec{KA} + mu , vec{KB} + vec{KC}big)\ &= frac{1}{3} big( lambda , vec{OA} + mu , vec{OA} + vec{OC} + (lambda + mu + 1) , nu , e_3 big)
        end{align*}
        The restrictions that lead to the equations for the unknowns $lambda, , mu$ and possibly $nu$ are
        begin{align*}
        |vec{C_qA_q}| &= |vec{A_qB_q}|\
        |vec{C_qB_q}| &= |vec{A_qB_q}|\
        vec{KG_q} & , |, | , vec{KG}\
        end{align*} where the first two say that the edges of equilateral triangle $A_qB_qC$ are equal while the last one represents the fact that the points $K, , G_q $ and $G$ are collinear. As equations, these three conditions are



        begin{align*}
        |lambda , vec{KA} - vec{KC}|^2 &= |lambda , vec{KA} - mu , vec{KB}|^2 \
        |lambda , vec{KA} - vec{KC}|^2 &= |lambda , vec{KA} - mu , vec{KB}|^2 \
        |vec{KG_q}times vec{KG}|^2 &= 0\
        end{align*}
        which written in terms of $lambda, , mu, , nu$ unknown variables turn into



        begin{align*}
        |lambda , vec{OA} - vec{OC} + (lambda , nu , - nu) ,
        e_3 |^2 &= |lambda , vec{OA} - mu , vec{OB} + (lambda ,
        nu , - mu , nu) ,
        e_3 |^2 \
        |mu , vec{OB} - vec{OC} + (mu , nu , - nu) , e_3 |^2 &=
        |lambda , vec{OA} - mu , vec{OB} + (lambda , nu , - mu
        , nu) ,
        e_3 |^2 \
        |big(frac{1}{3} big( lambda , vec{OA} + mu ,
        vec{OA} + vec{OC} + (lambda + mu + 1) , nu , e_3 big)big)
        times big(vec{OG} + nu , e_3 big) |^2 &= 0\
        end{align*}



        If you know $nu$ then only the first two equations are relevant



        begin{align*}
        |lambda , vec{OA} - vec{OC} + (lambda , nu , - nu) ,
        e_3 |^2 &= |lambda , vec{OA} - mu , vec{OB} + (lambda ,
        nu , - mu , nu) ,
        e_3 |^2 \
        |mu , vec{OB} - vec{OC} + (mu , nu , - nu) , e_3 |^2 &=
        |lambda , vec{OA} - mu , vec{OB} + (lambda , nu , - mu
        , nu) ,
        e_3 |^2
        end{align*}



        All these systems of equations may have several solutions though.



        I haven't checked for typos and mistakes and I do not guarantee this is the best solution.






        share|cite|improve this answer












        Let, as a model, we use a camera point $K$ and a camera screen $k$, where $k$ is a plane, and let the orthogonal projection $O$ of point $K$ on the plane $k$ be the origin of the two dimensional coordinate system $Oe_1e_2$ on the screen $k$. Attach a three dimensional coordinate system $Ke_1e_2e_3$ to the point $K$ where $e_1$ and $e_2$ are the unit vectors of $Oe_1e_2$ and let $e_3$ be the unit vector orthogonal to $k$ and pointing from $K$ to $k$. Denote by $nu$ the distance from $K$ to $k$. We may or may not know it.



        At first let $A_0B_0C_0$ be an equilateral triangle in a plane $p$ parallel to $k$. Let $G_0 in p$ be the center of $A_0B_0C_0$ so that $G_0$ lies on the $e_3$ axis, i.e. $G_0$ is the orthogonal projection of $K$ onto $p$. Then we project $A_0B_0C_0$ onto the plane $k$ by drawing the lines $KA_0, , KB_0, , KC_0$ and possibly $KG_0$ and denoting by $A, B, C$ and $O$ their intersection points with $k$. Thus we obtain the projected equilateral triangle $ABC$ with center $O$.



        Next we move the camera $K$ together with the screen $k$ to a different position and again we project the triangle $A_0B_0C_0$ with center $G_0$ onto the plane $k$ (which is now positioned differently with respect to $p$ and is this possibly not parallel to it) the same way we did before. We draw the lines between point $K$ and the points $A_0, B_0, C_0$ and $G_0$ and look at their respective intersection points $A, B, C$ and $G$ with plane $k$. The resulting triangle $ABC$ is most likely not equilateral.



        Think of the coordinate system $Ke_1e_2e_3$, which moved together with the camera and the screen, and recall that $nu$ is the distance between $K$ and $k$, the orthogonal projection of $K$ onto $k$ is $O$, and $Oe_1e_2$ is the coordinate system on the screen $k$ as before.



        If we take a plane $q$ parallel to $p$ passing through point say $C$, then the intersection points
        $$A_q = KA cap q, ,, B_q = KB cap q, ,, C_q = KC cap q , text{ and } , G_q = KG cap q$$
        form an equilateral triangle $A_qB_qC$ with center $G_q$ whose edges are parallel to the edges of $A_0B_0C_0$ (in other words triangle $A_qB_qC$ with center $G_q$ is homothetic image of triangle $A_0B_0C_0$ with center $G_0$ with respect $K$ )



        Now, you know the locations of the points $A, , B, , C$ and say $G$ with respect to the screen's $k$ coordinate system $Oe_1e_2$ which means you know the vectors $$vec{OA}, , vec{OB}, , vec{OC}, ,, text{ and } ,, vec{OG}$$ so thus, we can form the vectors
        begin{align*}
        vec{KA} &= vec{KO} + vec{OA} = vec{OA} + vec{KO} = vec{OA} + nu , e_3\
        vec{KB} &= vec{KO} + vec{OB} = vec{OB} + vec{KO} = vec{OB} + nu , e_3\
        vec{KC} &= vec{KO} + vec{OC} = vec{OC} + vec{KO} = vec{OC} + nu , e_3\
        vec{KG} &= vec{KO} + vec{OG} = vec{OG} + vec{KO} = vec{OG} + nu , e_3\
        end{align*}

        Thus, there exists constants $lambda, , mu$ and $nu$, if you do not know $nu$
        begin{align*}
        vec{KA_q} &= lambda , vec{KA}= lambda , vec{OA} + lambda , nu , e_3\
        vec{KB_q} &= mu , vec{KB}= mu , vec{OA} + mu , nu , e_3\
        vec{KG_q} &= frac{1}{3} big(lambda , vec{KA} + mu , vec{KB} + vec{KC}big)\ &= frac{1}{3} big( lambda , vec{OA} + mu , vec{OA} + vec{OC} + (lambda + mu + 1) , nu , e_3 big)
        end{align*}
        The restrictions that lead to the equations for the unknowns $lambda, , mu$ and possibly $nu$ are
        begin{align*}
        |vec{C_qA_q}| &= |vec{A_qB_q}|\
        |vec{C_qB_q}| &= |vec{A_qB_q}|\
        vec{KG_q} & , |, | , vec{KG}\
        end{align*} where the first two say that the edges of equilateral triangle $A_qB_qC$ are equal while the last one represents the fact that the points $K, , G_q $ and $G$ are collinear. As equations, these three conditions are



        begin{align*}
        |lambda , vec{KA} - vec{KC}|^2 &= |lambda , vec{KA} - mu , vec{KB}|^2 \
        |lambda , vec{KA} - vec{KC}|^2 &= |lambda , vec{KA} - mu , vec{KB}|^2 \
        |vec{KG_q}times vec{KG}|^2 &= 0\
        end{align*}
        which written in terms of $lambda, , mu, , nu$ unknown variables turn into



        begin{align*}
        |lambda , vec{OA} - vec{OC} + (lambda , nu , - nu) ,
        e_3 |^2 &= |lambda , vec{OA} - mu , vec{OB} + (lambda ,
        nu , - mu , nu) ,
        e_3 |^2 \
        |mu , vec{OB} - vec{OC} + (mu , nu , - nu) , e_3 |^2 &=
        |lambda , vec{OA} - mu , vec{OB} + (lambda , nu , - mu
        , nu) ,
        e_3 |^2 \
        |big(frac{1}{3} big( lambda , vec{OA} + mu ,
        vec{OA} + vec{OC} + (lambda + mu + 1) , nu , e_3 big)big)
        times big(vec{OG} + nu , e_3 big) |^2 &= 0\
        end{align*}



        If you know $nu$ then only the first two equations are relevant



        begin{align*}
        |lambda , vec{OA} - vec{OC} + (lambda , nu , - nu) ,
        e_3 |^2 &= |lambda , vec{OA} - mu , vec{OB} + (lambda ,
        nu , - mu , nu) ,
        e_3 |^2 \
        |mu , vec{OB} - vec{OC} + (mu , nu , - nu) , e_3 |^2 &=
        |lambda , vec{OA} - mu , vec{OB} + (lambda , nu , - mu
        , nu) ,
        e_3 |^2
        end{align*}



        All these systems of equations may have several solutions though.



        I haven't checked for typos and mistakes and I do not guarantee this is the best solution.







        share|cite|improve this answer












        share|cite|improve this answer



        share|cite|improve this answer










        answered Mar 13 '17 at 4:14









        Futurologist

        7,3062520




        7,3062520






























            draft saved

            draft discarded




















































            Thanks for contributing an answer to Mathematics Stack Exchange!


            • Please be sure to answer the question. Provide details and share your research!

            But avoid



            • Asking for help, clarification, or responding to other answers.

            • Making statements based on opinion; back them up with references or personal experience.


            Use MathJax to format equations. MathJax reference.


            To learn more, see our tips on writing great answers.





            Some of your past answers have not been well-received, and you're in danger of being blocked from answering.


            Please pay close attention to the following guidance:


            • Please be sure to answer the question. Provide details and share your research!

            But avoid



            • Asking for help, clarification, or responding to other answers.

            • Making statements based on opinion; back them up with references or personal experience.


            To learn more, see our tips on writing great answers.




            draft saved


            draft discarded














            StackExchange.ready(
            function () {
            StackExchange.openid.initPostLogin('.new-post-login', 'https%3a%2f%2fmath.stackexchange.com%2fquestions%2f2180102%2fhow-to-calculate-3d-rotation-from-3-points%23new-answer', 'question_page');
            }
            );

            Post as a guest















            Required, but never shown





















































            Required, but never shown














            Required, but never shown












            Required, but never shown







            Required, but never shown

































            Required, but never shown














            Required, but never shown












            Required, but never shown







            Required, but never shown







            Popular posts from this blog

            Plaza Victoria

            In PowerPoint, is there a keyboard shortcut for bulleted / numbered list?

            How to put 3 figures in Latex with 2 figures side by side and 1 below these side by side images but in...